LSAT and Law School Admissions Forum

Get expert LSAT preparation and law school admissions advice from PowerScore Test Preparation.

 ellenb
  • Posts: 260
  • Joined: Oct 22, 2012
|
#11875
So basically if we have

A-->B-->C-->F
or
D-->M

If we know A we cannot conclude C ? or not even F?
If we know A we cannot conclude D? or not even M?

or using the example from the stimulus.

where we have not Solved-->not more applicants-->Lower Standards
or ---> not High Quality Care
Shortage


What if we had:
where we have not Solved-->not more applicants-->Lower Standards-->not High Quality Care
or
Shortage

if we know that the problems have not been solved, than can we conclude not High Quality Care?

Thanks in advance!

Ellen
User avatar
 KelseyWoods
PowerScore Staff
  • PowerScore Staff
  • Posts: 1079
  • Joined: Jun 26, 2013
|
#11903
Hi Ellen!

I'm a little confused by how your diagrams ended up being aligned on the page but let me see if I can answer your questions.

If I understand your first diagram to be:

..... ..... ..... C :arrow: F
A :arrow: B :arrow: or
..... ..... ..... D :arrow: M

Then, you are correct. If you have A, you cannot conclude that you have C and F and you cannot conclude that you have D and M. You can only conclude that you have C OR D. So I could have A, B, C, and F or I could have A, B, D, and M.

So for the question, we know that we must have lower standards OR a shortage but we don't know for sure which one we will have. So answer choice (B) is wrong because not having more applicants would not guarantee that we must have lower standards. Maybe instead we just have a shortage. Likewise, answer choice (D) is wrong because not solving problems would not guarantee that we have a shortage. Maybe instead we have lower standards.

Hope this helps!

Best,
Kelsey
 ellenb
  • Posts: 260
  • Joined: Oct 22, 2012
|
#11920
So, to have a right answer they would have to put both:
If I have A than I might have C or D
and I cannot just say that I have C or that I have D. To make it correct, I need to say that if I have A than I can have C or D.

Ex:
lower standards
NOT solve problems --->NOT more applicants :arrow: or
..... ..... ..... ..... ..... ..... ..... ..... ..... shortage
 ellenb
  • Posts: 260
  • Joined: Oct 22, 2012
|
#11921
So, to have a right answer they would have to put both:
If I have A than I might have C or D
and I cannot just say that I have C or that I have D. To make it correct, I need to say that if I have A than I can have C or D.

Ex:
lower standards
NOT solve problems --->NOT more applicants------>or
shortage

Than if I have NOt Solve Problems than I have to include both lower standards or shortage in my answer, I cannot eliminate the other?

What if I have an and statement can I conclude than one without the other?
I believe this is a trick they do on the test, similar to where they left Mary out problem, with the teniss players.

thus if I have A-->B---> C and D

Then if I know A it is not necessary for me to put both C and D, the correct answer can be C. (unlike in an or statement where I have to have both) correct?
THanks

Ellen
 BethRibet
PowerScore Staff
  • PowerScore Staff
  • Posts: 200
  • Joined: Oct 17, 2012
|
#11931
Hi Ellen,

Thanks for writing in. You're on the right track. The critical thing to understand, which it seems like you're grasping now, is that when you have necessary conditions of which only one need be met (i.e. C OR D), then you can never conclude based on the presence of the sufficient condition which one of them is definitely present.

On the other hand, as you note from the prior tennis players question, if you have two necessary conditions which both must be true if a sufficient condition is met (i.e. C AND D), then an answer choice which says one of them is true and does not mention the other, provided the sufficient condition is met, is true.

Hope this helps!
Beth
 ellenb
  • Posts: 260
  • Joined: Oct 22, 2012
|
#14968
Dear Powerscore,

I had a question why answer choice B and D are not right?

If i diagram it

it should be something like:

For Answer choice D this is my though process


A--> D-> C or M


Thus, if we have A we can conclude C or M right? this is what the answers were trying to do.
PS not solved--> acute shortage (A--> M)

For answer choice B, it is like A-->C or M, and if we know A we can conclude M.

IF pull not increase--> than Lower entrance exams.

Please let me know whether my thought process is right and why those two answers are wrong, and whether I understood the answers and diagramed them correctly.

Regards,

Ellen
 Ron Gore
PowerScore Staff
  • PowerScore Staff
  • Posts: 220
  • Joined: May 15, 2013
|
#14974
Hi Ellen,

This is a Must Be True Question. The first sentence gives a conditional rule using the word "unless," which can be diagrammed as:

Sufficient ..... ..... :arrow: ..... ..... ..... Necessary

increase # applicants ..... :arrow: ..... ..... low wage/high stress problem solved

The next sentence tells us what happens if the applicant pool does not increase:

..... ..... ..... ..... ..... ..... ..... ..... ..... ..... lower entrance standards

increase # applicants ..... :arrow: ..... ..... or

..... ..... ..... ..... ..... ..... ..... ..... ..... ..... acute shortage of nurses

And, we are told, lowering the standards will not necessarily avert a shortage of nurses.

Finally, we are told that with either of the necessary conditions from the second sentence, "the current high quality of health care cannot be maintained."

In a Must Be True question that involves conditional reasoning, always be on the lookout for an answer choice that tests you on the contrapositive to the conditional relationships in the stimulus.

Here, the correct answer choice is (E). It says that if the problem of low wages and high stress (the necessary condition from the first sentence) are not solved, then the current high quality of health care cannot be maintained (i.e., the necessary condition from the last sentence). So, (E) is testing us on the contrapositive chain connecting the first sentence to the last sentence across the negated common term of "low wage/high stress problem solved."

Answer choice (B) is incorrect because it improperly states definitively that the profession will have to lower its entrance standards. This was only one of two possible necessary conditions. It was that the standards would have to be lowered or there would be an acute shortage of nurses. We cannot say with certainty which necessary condition will occur.

As with answer choice (B), answer choice (D) is incorrect because the "acute shortage of nurses" was only one of the two possible necessary conditions. It may be the case that the entrance standards are lowered. So, this is a could be true, rather than a must be true, answer choice.

Also, Ellen, I noticed after having written this response that you asked precisely the same question last year, and got detailed responses from Steve and Kelsey. Please be certain to check the archive prior to posting. If you've posted about the same question in the past, please mention that fact in your post, and let us know how we may be able to help you differently than we did in the past.

Thank you,

Ron
 ellenb
  • Posts: 260
  • Joined: Oct 22, 2012
|
#14975
Thanks Ron,

Just some follow-up questions, so how would you make/or what would make answer choices B and D right?

Also, we cannot diagram the following statement, and it is given just for extra information, since I know that on MBT questions, they tend to give extra information.

" it is not certain, however, that lowering entrance standards will avert a shortage"

Regards,

Ellen
 BethRibet
PowerScore Staff
  • PowerScore Staff
  • Posts: 200
  • Joined: Oct 17, 2012
|
#15019
Hi Ellen,

Thanks for writing.
Given the short time you have on each LR question, I wouldn't worry about how to make wrong answers correct. Mostly, it's to your advantage to just identify that they are not correct as written, and then move on!

Yes, you're correct that you don't need to diagram the sentence you quote, and it's not strictly essential to identifying the right answer choice.

best,
Beth
 Jalarcon
  • Posts: 4
  • Joined: Oct 15, 2014
|
#17095
I was wondering why is answer choice (d) incorrect? I diagrammed the stimulus as the following: ~PS-->~AGN-->LS-->~QM,
or
SN (or SN being directly under LS)

therefore wouldn't answer choice (d) essentially infer
~PS-->SN, which seems correct. (If problems not solved then there will be a shortage of nurses soon?) Please help!

Get the most out of your LSAT Prep Plus subscription.

Analyze and track your performance with our Testing and Analytics Package.